Solve the simultaneous equations:

6x-4y=39
2x+y=6

Answers

Answer 1
Answer:

Answer:

(9/2, -3)

Step-by-step explanation:


Related Questions

There are 49,000 seats in a sports stadium. Tickets for the seats in the upper level sell for $25, the ones in the middle level cost $30 and the ones in the lower level are $35 each. The number of seats in the middle and bottom levels together equal the number of seats in the upper level. When all of the seats are sold for an event, the total revenue is $1,419,500. How many seats have been sold in the upper level?
The Mathalot Company makes and sells textbooks. They have one linear function that represents the cost of producing textbooks and another linear function that models how much income they get from those textbooks. Describe the key features that would determine if these linear functions ever intercepted.
41 feet to yards, express the answer using a mixed fraction
Tickets to a concert cost $2 for children and $5 for adults. If 630 people attended the concert and the ticket sales totaled $2,910, how many children attended the concert?
How do you find the exact area of a circle having the given circumference of 3 pi

Enter the value of p so the expression 5/6 - 1/3n is equivalent to p(5-2n).

Answers

Let's solve for p when the two expressions are equal.

p(5-2n)=\frac{5}6 - \frac{1}3n  \n\n p=\frac{\frac{5}6-\frac{1}3n}{5-2n}*\frac{6}6= (5-2n)/(6(5-2n)) = \boxed{\frac{1}6}

Would any one please help me with this? My mom made a review thing for me to prepare for my midterm and I can't figure this one problem out. If someone could walk me through this I would be eternally grateful!

Answers


I ran into this same problem here on Brainly a couple of
weeks ago.  Only the drawing was posted, without asking
a question.  This one has some words with it, but it's not much
better than the other one was.

When you read it carefully, you can see blanks where there's
supposed to be something more written:

WHAT is equal to 3x + 9 ?

WHAT is equal to 8x + 11 ?

WHAT is equal to 5x - 8 ?

Find the  measures of WHAT, WHAT, and WHAT ?

You can't do a thing with this as it is.  It doesn't tell you
anything that you can work with.

If A is to the left of B on a number line, which of the following must be true?A.) A<B
B.) A=B
C.) A>B

Answers

Answer:

A.A<B

Step-by-step explanation:

We have been given that A is one the left of B on a number line. We are asked to choose the correct option from our given choices about the given condition.

We know that the value of numbers on number line increases from left to right.

Since A is on the left of B on number line, therefore, the value of a must be less than B and option A is the correct choice.

A.) A <B I hope i could help you.

If y − 3 = 3x, which of the following sets represents possible inputs and outputs of the function, represented as ordered pairs?A. {(0, 3), (1, 6), (2, 9)}
B. {(3, 0), (6, 1), (9, 2)}
C. {(1, 3), (2, 6), (3, 9)}
D. {(3, 1), (6, 2), (9, 3)}

Answers

B. {(3, 0), (6, 1), (9, 2)}

Answer:Ithink its A

Step-by-step explanation:

What are the possible rational zeros of f(x) = x4 + 6x3 – 3x2 + 17x – 15?.

Answers

f(x) = x^4 + 6x^3-3x^2 + 17x-15\n\n15:\{\pm1;\ \pm3;\ \pm5;\ \pm15\}\n1:\{\pm1\}\n\nAnswer:\boxed{\{\pm1;\ \pm3;\ \pm5;\ \pm15\}}

A box plot was made to represent the number of matches won by 14 participants in a tennis tournament. The box plot had the box shifted to the right so that the left tail was much longer than the right tail. Based on the box plot, which conclusion is correct? (A)The mean and median of matches won are equal.
(B)The mean of matches won is more than the median of matches won.
(C)Most of the participants won many matches, but there were some participants who won very few matches compared to the others.
(D)Most of the participants won very few matches, but there were some participants who won many matches compared to the others.

Answers

Answer:

(C)- Most of the participants won many matches, but there were some participants who won very few matches compared to the others.

Step-by-step explanation:

The Box plot isshifted to the right this means it is right-skewed data thus left tail is much longer than right tail. Hence, Option (C) is correct.

Option (A) means than neither right tail is longer nor left, the box will remain in middle only. So it is incorrect.

Option (B) is incorrect because Box Plot doesn't give us idea about mean.

Option (D) is also not correct because if this happened then box will shifted to the left

The box plot had the box shifted to the right so that the left tail was much longer than the right tail. Based on the box plot, the conclusion that is correct is Most of the participants won very few matches, but there were some participants who won many matches compared to the others. The answer is letter D